Download as pdf or txt
Download as pdf or txt
You are on page 1of 42

lOMoARcPSD|3248894

Tutorial work - weekly tutorial problems

Commercial Law (RMIT)

StuDocu is not sponsored or endorsed by any college or university


Downloaded by ahmad as (ksa-m29@hotmail.com)
lOMoARcPSD|3248894

RMIT
COMMERCIAL LAW
LAW2442

Weekly Tutorial Problems

Semester 1, 2015

Downloaded by ahmad as (ksa-m29@hotmail.com)


lOMoARcPSD|3248894

Week 2 (CH1) Introducing the Law

1. How would you define the law? A system of rules made by the state and
enforceable by prosecution or litigation (page 8 of text)

2. In what ways does the law impact upon your personal life? Various ways.
Contract law is buying a coffee in the morning, Tort law governs a doctor who
negligently performs a procedure, Buying a house involves Property law
3. In what ways does the law impact upon business activities? Paying tax,
Licencing, Advertising, OH&S
4. What is the difference between public law and private law?
Public Law concerns the relationship between the citizen and the state (tax law,
crim)
Private Law concerns the relationship between individuals (contract law, tort
law)

5. Which category or categories of law would be the most relevant in each of


the following situations?
(a) The Federal Parliament has passed legislation establishing new
industrial relations regime. The Queensland government claims that the
Federal Parliament does not have authority to pass such a law.
Constitutional
(b) A journalist has written a news story that contains defamatory allegations
about Ash that are not true. Tort law - Defamation
(c) The Department o f Transport has refused t o grant Johnny's mother a
driver’s licence. Johnny’s mother believes that the decision maker was
biased. Administrative law
(d) Ash has been charged with shoplifting. Criminal Law
(e) One of Johnny's competitors is using a logo that looks substantially
similar to Johnny's logo. Copyright law
(f) In breach of a written agreement, one of Johnny's suppliers has failed to
deliver important supplies to his restaurant on the agreed date.
Contrac law.

6. What are the three levels of government in Australia? Fedaral Gov, State gov,
Local Council.
7. What is the judiciary and what is its role? Judiciary: The courts
Their role is to make law through case decisions but also interpret statute law

8. Why is the law often changing? Political change, Need to fix a problem,
Changing community values, Pressure from loby groups, Changing tech.

Downloaded by ahmad as (ksa-m29@hotmail.com)


lOMoARcPSD|3248894

3
Week 3 (CH2 and CH3)
(a)Understanding the Australian Legal System
(b)Exercising Legal Skills
1. Distinguish between:

(a) Common law courts and courts of equity


Common law is law made by the courts in accordance with the doctrine of
precedence.
Equity is a category of case law rules and remedies based on fairness and
justice, developed to supplement the common law (pg 103)
Equity developed because of perceived and actual rigidities and injustices
in the common law
3 importance creations from equity: Specific performance, Injunction,
Trust
(b) Common law and statutory law
Common law is made by courts; Statutory law is mae by parliment
(c) Common law and civil law systems of law
Common law system: Adopted in Victoria and is based on two sources of law;
Legislation and case law
Civil law system: Based on the French system where the entire law is written
down and enacted as a series of very detailed statutes (codification)
(d) Criminal law and civil law
Criminal law: Looks at crimes against society as a whole.
Civil law: Individuals enforcing their rights.
(e) Ratio decidendi and obiter dicta.
Ratio decidendi: (reason for the decision). The part of a judges decision that set
out the legal principle upon which the final decision is made.
Obiter dicta: (saying: BTW). The part of the judges decision other than the legal
principle upon which the final decision is based.

Downloaded by ahmad as (ksa-m29@hotmail.com)


lOMoARcPSD|3248894

2. Study the following case extract:

AUSTRALIAN SAFEWAY STORES PTY LTD v ZALUZNA

HIGH COURT OF AUSTRALIA


MASON, WILSON, BRENNAN, DEANE and DAWSON JJ
36 November 1986, 10 March 1987 - Canberra

Negligence - Duty of care - Duty of occupier to invitee - Occupiers liability -


Personal injuries - Plaintiff slipped on wet floor - Whether special duty of care
owed to invitee - Whether general duty of care.

The respondent was injured when she slipped in the appellant's supermarket on a
floor which had become wet as a result of persons entering from the rain outside. The
trial judge applied the statement of the duty of care owed by an occupier to an invitee
formulated in Indermaur v Dames (1866) LR 1 CP 274 and found that, it being a
rainy day, the moisture on the floor did not constitute an "unusual danger" within that
formulation. Accordingly, he dismissed the respondent's action.

The respondent appealed on the ground, inter alia, that the trial judge should have
considered whether the appellant owed a general duty of care to the respondent in
addition to the special duty of care owed
to an invitee. The Full Court of the Supreme Court of Victoria allowed the appeal on
this ground and an appeal from that decision was taken to the High Court.

Held, per Mason, Wilson, Deane and Dawson JJ, dismissing the appeal:

The duty which an occupier of land owed to an invitee was properly to be seen as the
ordinary common
law duty to take reasonable care.

Hackshaw v Shaw (1984) 155 CLR 614; 56 ALR 417; Papantonakis v Australian
Telecommunications Commission (1985) 156 CLR 7; 57 ALR 1, followed.
Indermaur v Dames (1866) LR 1 CP 274, explained.
London Graving Dock Co Ltd v Horton [1951] AC 737, disapproved.

What was reasonable would vary with the circumstances of the plaintiff's entry upon the
premises. Observations on the nature of the so-called special duties resting on an
occupier of land with respect to persons entering as licensees or trespassers and in
other circumstances.

San Sebastian pty Ltd v Minister Administering the Environmental Planning and
Assessment Act 1979 (2986) 68 ALR 161; Cook v Cook (1986) 61 ALJR
25; 68 ALR 353, referred to.

Appeal
This was an appeal from the order of the Full Court of the Supreme Court of
Victoria allowing an appeal from the decision of a single judge of the court and
ordering a new trial. The trial judge had dismissed the plaintiff's action.

J E Barnard QC and J G Meagher for the appellant.


D J Ashley QC and P F O'Dwyer for the respondent.

Downloaded by ahmad as (ksa-m29@hotmail.com)


lOMoARcPSD|3248894

5
Now answer the following questions:

(a) Which court is this decision reported from?


High court of AU

(b) Is this a matter heard at first instance?


No, it is an appeal from the full court of the supreme court of Victoria (now
court of appeal) which in turn was an appeal from a single judges of that court.

(c) Who is appellant? Who is the respondent?


A: Australian Safeway stores.
R: Mrs Zaluzna

(d) Where in the report are the catchwords?


Negligence - Duty of care - Duty of occupier to invitee - Occupiers
liability - Personal injuries - Plaintiff slipped on wet floor - Whether special
duty of care owed to invitee - Whether general duty of care.

(e) Where in the report is the headnote?


Immediately below the catchwords and ending before the word ‘appeal’ in dark
print. It summarizes the facts, issues and decision in the case and its progress
through any lower courts if it is an appeal

(f) What was the decision of the court?


The appeal was dismissed. Mrs.Z won the appeal

(g) Which judge(s) comprised the court?


Mason, Wilson, Brennan, Deane and Dawson JJ

(h) Is this a civil or a criminal matter? Civil

3. What is the doctrine of precedent? What are law reports?


Doctrine of precedent: The principle that when deciding a question of law a court
must do so consistently with the earlier decisions of higher courts within the
court hierarchy. There are binding and persuasive precedents
Law reports: are Published court judgments.

4. The Australian Constitution gives the Australian government full power to make
any laws it wants. Discuss this statement.
The commonwealth parliament does not have the power to make any laws it
wants
It has exclusive powers and headsof power under s51 of constitution
S51 powers are also concurrent with the states.
If there is inconsistency, commonwealth law will prevail: section 109 of
constitution

5. Which government (State or Federal) can pass legislation in respect of the


following matters?

(a) Education: State power


(b) Trade and commerce: Federal
(c) The Royal Australian Air Force: Federal
(d) Taxation: Fed and state
(e) Duties of customs and excise: Fed
(f) Environmental issues: State

Downloaded by ahmad as (ksa-m29@hotmail.com)


lOMoARcPSD|3248894

6
Week 4 (CH4) Causing Harm
1. What is a "tort"? How does it differ from a "crime"?
A tort is a civil wrong. Tort law is concerned with compensating a victim for the
harm caused to them by another outside contract law
A crime is concerned with punishing the wrongdoer either by fine or punishment

2. Distinguish between trespass, detinue, battery, assault, nuisance, defamation and


negligence.
Trespass: where a person intentionally or negligently interferes with the person
or property of the plaintiff.
Detinue: Type of trespass to goods where the defendant lawfully has
compensation of the plaintiff’s goods but refuses to return them.
Battery: Direct contact with the plaintiff’s body without the latters consent or
lawfull justification.
Assault: The threat of physical contact with the plaintiff
Nuisance: Indirect interference with the plaintiff’s use nd enjoyment of private or
public land
Defamation: Spoken if written publication od statements damaging to the
plaintiffs reputation.
Negligence: Where a careless act causes harm to the plaintiff.

3. When is an employer liable for the conduct of an employee?


Principle of vicarious liability (pg 167), provides an employer will be held
responsible for the actions of their employee.
Applies where employee is undertaking authorised work in an unathorised or
wrongful manner (Century Insurance Co v Northern Ireland Road Transport).
Will not apply if acting outside of scope of employment (Deatons v Flew).

4. Johnny sees George, a young customer at his restaurant, reach into the open
cash register and grab a handful of notes while Stephen, the person serving
him, is looking the other way. Johnny rushes up to George, grabs his arm,
pulls the money out of his hand, pushes him into a chair, and orders him to
'stay there until the police arrive, or I will lose my temper'. Thirty minutes later
the police arrive and take George away for questioning. George has now
threatened to sue Johnny in the tort of trespass to the person. Has Johnny
committed the tort? When answering this question consider all three forms of
the tort.

3 types of trespass to the person: Battery, Assault, False imprisonment.

Has johnny commited battery : Probably yes.

Downloaded by ahmad as (ksa-m29@hotmail.com)


lOMoARcPSD|3248894

Assault : Probably yes

False imprisonment : Probably yes


Defences:
Defence of property: - Tort of battery
Citizens arrest: Where a private person may arrest another person if there are reasonable grounds to
suspect that the other person is committing or has committed an offence.

4. Elaine has opened a new steakhouse restaurant next door to Johnny's vegan
restaurant. Johnny is concerned that the smell of cooking meat wafting from
the steakhouse is turning customers away from his restaurant. Can he establish
that Elaine is committing the tort of private nuisance? If so, what remedy would
Johnny be entitled to?

• The main issue is whether Elaine’s interference is ‘unreasonable’?


• Her restaurant is in a strip of restaurants, and most restaurants cook meat,
the smell is therefore not unreasonable
• However, if Johnny is successful, he can claim damages for loss of
customers & injunction prohibiting Elaine cooking meat.

6. Robin writes a popular restaurant review blog for the local online city guide.
Her latest blog includes the following: 'By the way, I ate at a certain vegan
restaurant on Kerouac Avenue last Saturday night. Whatever you do, do not
order the tofu burger unless you happen to like eating something that tastes
like the congealed leftovers of a liposuction treatment that is smeared between
two slices of stale bread.' Johnny wants to sue Robin in the tort of defamation. Is
Johnny's action likely to succeed? When answering this question consider:

Downloaded by ahmad as (ksa-m29@hotmail.com)


lOMoARcPSD|3248894

8
(a) the three elements of the tort, and

(b) Whether or not Robin can rely upon the defence of honest opinion.

• It seems Robin has satisfied all elements, therefore Johnny is entitled to


damages & injunction
• However, Robin may claim defence of honest opinion which requires:
1. Statement is opinion, not fact
2. Made in relation to a matter of public interest
3. Based on material substantially true
4. Is honestly held
Therefore it seems the defence of honest opinion has been satisfied and Johnny
will not be entitled to damages and an injunction.

7. What elements do you need to prove to make a case of negligence? What


defences are available?

• Defences:
– Voluntary assumption of risk
– Contributory negligence

Downloaded by ahmad as (ksa-m29@hotmail.com)


lOMoARcPSD|3248894

Week 5 (CH4) Causing Harm


Negligence and Negligent Misstatement
1. Outline the elements necessary to prove to succeed in the tort of negligence.
1. The defendant owes the plaintiff a duty of care
2. The defendant breached their duty of care
3. The breach caused harm / loss to the plaintiff
2. How do we test whether one person owes another person a duty of care?
a. Does the relationship provide for an existing duty?
b. If not, look at
i. reasonable foreseeability (Donoghue v Stevenson; Bourhill v
Young)
ii. features of the case compared with other cases (Sullivan v Moody)

3. Sinh is a student at RMIT University. The female toilets at RMIT University have
recently been cleaned and when Sinh uses the toilets she slips on a very wet floor
in the wash room and breaks her leg. As a result of the accident she has incurred
significant medical expenses and is out of her part time work for four months.
(a) What legal action could Sinh take against RMIT University or the cleaners
of the toilets and how likely will she be to succeed?

ISSUE: The legal issue in this question is whether Sinh can take negligence proceedings
against RMIT.
RULE: To succeed, Sinh must establish that:
(a) A duty of care is owed (Donoghue v Stevenson)
(b) The duty of care has been breached (Imbree v McNeilly)
(c) The breach caused the damage (Chappel v Hart; Wagon Mound case)
APPLY: Apply the law to the facts – class discussion
CONCLUSION: Based on the preceding analysis, it is more likely that Sinh will be
successful in suing RMIT in negligence
(b) Is the situation different if Sinh suffered her injury after breaking into RMIT
University when the University is closed with the intention of stealing a
Commercial Law examination?
Perhaps:
*In some circumstances, a duty of care is also owed to uninvited visitors such as
trespassers.
*In Hackshaw v Shaw, it was held that the defendant farmer owed a duty of care to a
trespasser to avoid “ultra-hazardous” acts (firing bullets).
*However, in Bryant v Fawdon the occupier was held not to owe a duty of care to the
plaintiff trespasser, given that the injury was not reasonably foreseeable (the
plaintiff climbed a 1.8 metre fence to get to a toilet and was knocked unconscious
when she attempted to flush the toilet’s disused concrete cistern in the dark).

4. Cameron was attending a rock concert in the park. He had not purchased a ticket
but had climbed over a two metre fence to join the spectators who had purchased
tickets. Strong wind and heavy rain forced the spectators to take cover from a
storm under a temporary tarpaulin set up by the organisers to protect the
musicians and their equipment. During the storm the tarpaulin became heavy with
water and collapsed onto the musicians, their equipment and spectators who had
taken shelter under it. Cameron's back was seriously injured. As a result of the
accident he required emergency surgery. Cameron was unable to complete his
university studies that year. Another spectator standing nearby, Alec, saw
Cameron have his back crushed and consequently suffered psychological distress.
Downloaded by ahmad as (ksa-m29@hotmail.com)
lOMoARcPSD|3248894

10

Advise both:

(a) Cameron; and

ISSUE: Is the rock concert organiser liable in negligence to Cameron?

RULE: To succeed, Cameron must establish that:


(a) A duty of care is owed (Donoghue v Stevenson)
(b) The duty of care has been breached (Imbree v McNeilly)
(c) The breach caused the damage (Chappel v Hart; Wagon Mound case)

APPLY:
Duty of care: It appears reasonably foreseeable that people would seek
shelter, and it would collapse, may injure: Donoghue v Stevenson; Safeway v
Zaluzna; Hackshaw v Shaw; Bryant v Fawdon
Breach: Yes, because it is highly probable people would be injured if
tarpaulin collapses, and burden of eliminating risk not very difficult or
expensive.
Damage: Cameron’s injuries were caused by breach and not too remote.

DEFENCES: Voluntary assumption of risk (N/A); contributory negligence possible

CONCLUSION: The rock concert organiser is liable in part to Cameron.

(c) Alec

ISSUE: Is the rock concert organiser liable in negligence to Alec?

RULE: The same rules apply as previously discussed in (a)

APPLY: Alec has an argument that it is reasonably foreseeable a witness could sustain
harm: Bourhill v Young (nervous shock – no duty of care); Chapman v Hearse (car accident
– duty owed to passers by who stopped to help)

CONCLUSION: Rock concert organiser is liable in negligence

5. You are at your office Christmas party. Whilst speaking to your boss you start to
choke on the rice ball hors-d'oeuvre you are eating. You look down and realise
you have been eating half a mouse which is inside the 'rice ball'. You immediately
feel sick and contract severe food poisoning. You are rushed to hospital and whilst
recovering in hospital you fall down the hospital stairs on your way to visit the
hospital garden. Consequently you break your arm in the fall and have to stay in
hospital for an extra one month incurring further medical expenses.

Evaluate whether you would be likely to receive compensation in a negligence


claim against the caterers at the Christmas party for both
Downloaded by ahmad as (ksa-m29@hotmail.com)
lOMoARcPSD|3248894

11

(a) the food poisoning; and

ISSUE: The legal issue in this matter is whether the caterers are liable in negligence to
compensate you for food poisoning.

RULE: To succeed, you must establish that:


(a) A duty of care is owed (Donoghue v Stevenson)
(b) The duty of care has been breached:
- Probability of harm (Bolton v Stone)
- Seriousness of harm (Paris v Stepney Borough Council)
- Burden of taking precautions (Latimer v AEXC Ltd)
(c) The breach caused the damage:
- Factual causation
- Scope of liability
(Chappel v Hart; Wagon Mound case)

• Defences ?

APPLY: Apply the law to the facts – class discussion

CONCLUSION: Based on the preceding analysis you are likely to be successful against
the caterers for food poisoning
(c) your broken arm.
You are unlikely to recover compensation for this – too remote

6. The tort of negligent misstatement requires that in order for a duty of care to arise
between persons, there needs to be a 'special relationship'.

(a) What are the factors a court will have regard to in determining whether such
a relationship exists?
• MLC v Evatt (1968) established that a professional adviser owed a duty of
care to clients to whom they supplied information.
• In L Shaddock and Associates Pty Ltd v Parramatta City Council (1981) the
duty was widened to include any persons supplying information to others
where they knew that the information would be relied upon for a serious
purpose.

The following requirements must be satisfied:


• The advice was of a business or serious nature
• The defendant knew or should have known that the plaintiff intended to rely
on the advice.
• It was reasonable in the circumstances for the plaintiff to rely on the
defendant’s advice.

(b) Do you agree with the statement that "where a professional relationship
exists (for example; accountant and client, lawyer and client, marketing
executive and client) there is a duty of care owed by the professional to
their client"?
If the professional gives careless advice that leads to economic loss then the
cases referred to in question 6(a) support that professional being held
accountable.

Downloaded by ahmad as (ksa-m29@hotmail.com)


lOMoARcPSD|3248894

12
7. You are an accountant in practice. It is the 29 June 2015. You are busy doing
outstanding tax returns for your clients. A client telephones you to ask 'what is the
maximum deductible contribution he can make to superannuation for the year
ended 30 June 2015?" You tell him $15,000. However you are incorrect and the
amount is $25,000 because of his age. As a result of your error the client needs to
pay additional tax of $2,500.

Are you, the accountant, negligent for your advice?

• ISSUE: The legal issue in this matter is whether you are liable in your capacity
as an accountant for providing incorrect advice and causing economic loss.

• RULE: To succeed you must satisfy:


• The accountant owed a duty of care;
• The accountant breached their duty;
• The accountant’s breach caused economic loss.

In negligent misstatement actions, the duty of care is differently ascertained.


Three criteria are required to be satisfied:

• The advice was of a business or serious nature


• The defendant knew or should have known that the plaintiff intended to rely on
the advice.
• It was reasonable in the circumstances for the plaintiff to rely on the
defendant’s advice.

• A duty of care in negligent misstatement claims arises where there is a


“special relationship” between the parties (Hedley Byrne & Co Ltd v Heller and
Partners Ltd [1964] )
• The test for establishing a “special relationship” was set out in (L Shaddock
and Associates Pty Ltd v Parramatta City Council (1981))
• APPLY:
• CONCLUSION:

Downloaded by ahmad as (ksa-m29@hotmail.com)


lOMoARcPSD|3248894

13

8. Oanh inherited $50,000 from her aunt and decided to invest the money in rural
property. After considerable searching for a suitable site, she considered buying
500 acres of land being offered within the Meadows Shire Council's administrative
area. Oanh was unaware of the block's zoning status.

While investigating the suitability of the land for the purpose of establishing a
boutique vineyard. She approached the Meadows Shire Council for advice and
information as to the possible establishment of a future vineyard on the land. She
was advised by Suresh, a land surveyor employed by the Council, that the property
was "zone two and suitable for agricultural use". In their conversation, Suresh told
Oanh that there were no notices or restrictions in force in regard to the area which
would have the result of changing this zoning or having any impact upon the
land or its future use. In providing this advice Suresh failed to notice and take
account of a restriction attached to the Council area map warning that the land
was subject to a State Government order that all land in zone two was to be used
for residential purposes only.

Oanh bought the land and employed a team of viticulturalists to design and
establish a vineyard. Shortly after the grape vines were planted Oanh received a
notice from the State Government requiring that her land be only used for
residential purposes. Advise Oanh

• The facts give rise to a claim in negligent misstatement.


• An action for economic loss caused by negligent misstatement may succeed if the
following can be established: (1) duty of care, (2) breach in the requisite standard of
care and (3) damage (caused by the breach and which is reasonably foreseeable).
• Defences should also be considered.

• ISSUE: The main issue is whether the Meadows Shire Council (MSC) owed Oanh a
duty of care.
• RULE: To succeed you must satisfy:
• MSC owed a duty of care;
• MSC breached their duty;
• MSC breach caused economic loss.

In negligent misstatement actions, the duty of care is differently ascertained.


Three criteria are required to be satisfied:

• The advice was of a business or serious nature


• The defendant knew or should have known that the plaintiff intended to rely on
the advice.
• It was reasonable in the circumstances for the plaintiff to rely on the
defendant’s advice.

• APPLY:
• CONCLUSION:

Downloaded by ahmad as (ksa-m29@hotmail.com)


lOMoARcPSD|3248894

14
9. Andrew Garrett, a well-known radio investment advisor was cornered by Victoria
Spence, his neighbour, whilst putting out his rubbish bins early one morning.
Victoria, in her dressing gown, asked Andrew if he considered the company ASA
Pty Ltd to be a company with bright prospects. Andrew replied "yes" and promptly
left Victoria to return to inside his house. The next day Victoria bought $10,000
worth of shares in ASA Pty Ltd. A week later ASA Pty Ltd went into liquidation and
Victoria learned that she would be highly unlikely to recover her $10,000.

Victoria seeks your advice as to whether she has a cause of action against
Andrew. Advise her accordingly, giving reasons for your advice.

Would it have made any difference to your advice had Victoria telephoned Andrew
for his investment advice whilst he was on the radio?

Downloaded by ahmad as (ksa-m29@hotmail.com)


lOMoARcPSD|3248894

15

Week 6 (CH5): Making Deals: Forming a Contract


1. A contract creates a set of legally enforceable obligations and rights. Describe in
your own words what the effect of this means.
 This statement means that a contract can be enforced in a court of law if the
other party fails to comply with their contractual obligations.
 If a party is misled or suffers a loss, they are able to seek compensation.

2. What are the requirements or elements that 'form' a contract or bring a contract
into existence?
In order for a contract to be legally enforceable, it must adhere to certain
requirements:
• there must be an agreement;
• the parties must intend their agreement to be enforceable; and
• Both parties must provide consideration.

3. Describe the second requirement or element that is necessary to turn an


agreement/arrangement or understanding into a legally binding contract? In other
words, what is the second legal requirement to form a contract and what are the
two main presumptions that arise in relation to this requirement?
 The second requirement is ‘intention’. The parties must intend their
agreement to be legally binding.
 Social and domestic agreements are presumed not to be binding (Balfour v
Balfour [1919] 2 KB 571).
 Commercial or business agreements are presumed to be binding. These
presumptions can be rebutted by evidence to the contrary (Wakeling v Ripley
(1951) 51 SR (NSW) 183).

4. Ada lived in Singapore and sent a series of emails to her niece, Chloe, who lived
in Australia. Ada's emails encouraged her niece to move to Singapore telling
Chloe "you can live in my house and pay no rent if you come and keep me
company". Ada even made promises to alter her will to provide that Chloe could
live in Ada's house for life free of rent. Chloe accepted the offer, quit her job as a
marketing executive, sold her house in Australia and flew to Singapore. After
arriving, Ada and Chloe had a fight and Ada asked Chloe to leave the house.
Advise Chloe using common law principles.
ISSUE: The legal issue in this matter is whether there is an intention to be legally bound in
the agreement between Chloe and Ada.
RULE: In family or domestic arrangements, there is a presumption in law that there is no
intention to be legally bound: Balfour v Balfour. However this is a rebuttable presumption.
APPLY: The case bears a remarkable similarity to Wakeling v Ripley. Both cases involve
plaintiffs who sell up and shift locations to move in with older relatives on promises to
inherit.
Here, Chloe quit her job as a marketing executive and sold her house to live with her
aunt Ada in Singapore rent free.
CONCLUSION: The facts here indicate that there is a contract between Chloe and Ada. The
presumption that there is no intention to be legally bound can be rebutted because of
serious economic consequences.

Downloaded by ahmad as (ksa-m29@hotmail.com)


lOMoARcPSD|3248894

16
5. Nadya runs an accounting practice. In the practice shop window is a sign that
reads “Annual Taxation Returns for $100". Nadya, however, has recently
increased the price for this service to $120. Gideon is walking past Nadya’s
practice and enters. Gideon says to Nadya "I would like you to do my tax return for
the price in the window". Is Nadya contractually obligated to do Gideon's tax
return for $100?
ISSUE: The legal issue in this matter is whether the shop sign “Annual Taxation returns for $100”
is an offer or invitation to treat.

RULE: An offer is a clear statement of the terms on which an offeror is prepared to be


contractually bound. An offer can be contrasted with an invitation to treat which is an invitation to
another person to make an offer.

APPLY: In this scenario the sign “Annual Taxation returns for $100” appears to be an invitation to
treat because it is a sign displayed in a shop window. This is consistent with the ruling in
Pharmaceutical Society (GB) v Boots Cash Chemists which held that displays in a shop are an
invitation to treat. Therefore Nadya can argue that her sign is an invitation to treat which invites
anyone walking past to make an offer. Nadya can then ultimately accept or reject this offer.

CONCLUSION: Based on the preceding analysis, Nadya will not be bound to do Gideon’s tax
return for $100.

6. Dewi and Cheng are good friends. Dewi heard that Cheng was keen to buy his
car. Dewi telephoned Cheng and said "I hear you want to buy my Honda. You
can have it for $3,000. I'll give you a week to make up your mind".

The next day Dewi met another friend Ethan and after some discussion sold Ethan
the Honda car for $3,500. Two days later Cheng heard from a mutual friend Pia
that Dewi had already sold the car. Cheng immediately rang Dewi to accept the
Honda for $3,000.

Does Cheng have a concluded contract with Dewi? Would your answer be
different if Cheng had given Dewi $50 to keep the offer open?

ISSUE: The legal issue in this matter concerns intention, agreement and options.
RULE:
Agreement?
Intention- Is there an intention to be legally bound? Balfour v Balfour; Wakeling v Ripley
Consideration?
Option

APPLY:
• Commercial agreement, intention satisfied.
• Dewi makes an offer but without $$ it can be revoked any time
• When Cheng hears car has been sold, it implies the car is no longer for sale and
amounts to effective revocation
• If Cheng had paid $50 to Dewi to keep offer open, that would have been an option
which makes offer irrevocable: Goldsborough Mort v Quinn

CONCLUSION: There is no binding contract between Cheng and Dewi. If option had been paid
for, Cheng could sue Dewi for damages.

Downloaded by ahmad as (ksa-m29@hotmail.com)


lOMoARcPSD|3248894

17

7. You are an accountant/adviser and you need a new mobile phone. You walk into
the Telstra shop on the corner of Swanston and Bourke Street Melbourne. The
salesperson tells you to buy the latest Samsung Galaxy model. It is better than the
Apple I Phone coming out soon and cheaper. You are not convinced and speak to
your friend. He tells you that his sister, Ivy, wants to sell her Samsung Galaxy
phone. You send Ivy an email "I hear that you want to sell your Samsung Galaxy.
If it is in good condition and if the price is right, I would like to buy it. Please advise
me as soon as possible". Ivy replies by email "The phone is in excellent condition
and is cheap at $700". You email Ivy back and say "I accept your offer and will buy
the phone for $700". However before reading your email Ivy receives an email
from another person who offers her $800 for the phone. Ivy sells the phone to the
other person for $800.

(a) Is there a contract between Ivy and you for the phone?
ISSUE: The legal issue in this matter concerns the contractual elements of ‘offer', ‘acceptance’
and the ‘postal acceptance rule’.

RULE:
• There must be clear offer & acceptance: Powell v Lee
• Postal acceptance rule: Adams v Lindsell
• If using email, postal acceptance rule does not apply. See Electronic Transactions
Act 1999 (Cth)
APPLY:
Statement lacks the quality of a definite undertaking to be bound. More likely to be request
for information: Harvey v Facey.
If Ivy’s response is deemed to be an offer, more likely there is a contract. Acceptance sent
via email deemed to be ‘received’ when it becomes capable of being retrieved by Ivy.

CONCLUSION:
Based on the preceding analysis, there is a valid agreement between Ivy and you, only if
Ivy’s response is considered an offer. Otherwise, no contract has been formed.

(b) How might your answer be different (if at all) if the conversation between Ivy
and you by email was as follows:

"Hi Ivy, I would like to buy your Samsung Galaxy for $700, please let me
know by email". Two days later you change your mind and purchase the
latest Apple I Phone from a shop. The same day you receive an email from
Ivy which reads "Great. It's yours".

Is there a contract between Ivy and you?


In these circumstances your email may be deemed an offer. However, it seems by
purchasing an Apple iPhone you intend to revoke your offer. Revocation is not
effective until communicated (Byrne v Van Tienhoven) to the offeree. Accordingly
Ivy’s acceptance is valid and a contract has been formed.
(c) Would your answer to (b) above be any different if you chose not to read
Ivy's email but instead delete it?
As Ivy and you conducted your communication by email, it can be implied that your
email addresses were specified. Thus, under s 14A(1) of the Electronic
Transactions Act 1999 (Cth), Ivy’s email accepting your offer is considered to have
been received by you when it became capable of being retrieved by you. It is
therefore irrelevant that you deleted Ivy’s email of acceptance rather than reading it.
You are still deemed to have received the acceptance, and a contract will therefore
have been formed.
Downloaded by ahmad as (ksa-m29@hotmail.com)
lOMoARcPSD|3248894

18
8. How would you describe consideration? Giving examples from case-law, what
different forms can consideration take or not take?
• Consideration is the price paid for the other party’s performance of a
contract. Consideration can take many forms including payment of money,
provision of goods, provision of a service, and undertaking of an onerous
obligation or refraining from doing something.
• In Hamer v Sidway an uncle promised his nephew money if the nephew
promised not to gamble, smoke or drink until he turned 21. The uncle died
and the executor refused to pay. The court held forbearance by the nephew
was good consideration.
• Consideration does not need to be adequate (Thomas v Thomas (1842)) but it
does need to be sufficient (i.e. it must have some legal value).
• Consideration cannot be too vague: White v Bluett (1853); Placer
Development Ltd v Commonwealth (1969)
• Consideration will not be acceptable if it is ‘past consideration’ (Roscorla v
Thomas) or if it is for an existing contractual obligation: Stilk v Myrick (1809)

9. Saanvi's father, Mohammed, promises Saanvi that if she studies hard and obtains
a high distinction for her Commercial Law course at RMIT, he will give her $1,000.
Saanvi and Mohammed sign a written agreement to this effect. Saanvi spends all
her free time studying Commercial Law and receives a high distinction. Saanvi
asks her father for the money he promised but he refuses saying "you haven't
done anything that you're not already required to do. Your high marks should be
enough reward".
Can Saanvi take successful legal action against her father?

ISSUE: The legal issues in this matter involve intention and consideration.
RULE: Social and domestic agreements are presumed not to be binding (Balfour v Balfour). These
presumptions can be rebutted by evidence to the contrary (Wakeling v Ripley).

Consideration will not be acceptable if it is for an existing contractual obligation: Stilk v


Myrick (1809)

APPLY: Family or domestic arrangements are presumed to not have an intention to be legally
bound but this can be rebutted.

Mohammed could argue Saanvi is doing no more than what was already expected to do.
However, Saanvi could argue she did not have prior obligation to get HD. She could fail,
pass or receive any other outcome.

CONCLUSION: Saanvi may be successful in taking action against her father to enforce the
agreement.

Downloaded by ahmad as (ksa-m29@hotmail.com)


lOMoARcPSD|3248894

19

10. Katie is employed by a business solutions firm. She is working in a team of four
persons to complete a special project by 1 August 2015. Two of the team
members leave the firm, leaving the special project short staffed. Katie's boss
promises Katie that if she works weekends on the project, he will pay her an extra
$500 per weekend. When the project is completed, Katie's boss refuses to pay
the $500 for each of the three weekends she has worked.

Advise Katie whether she has any rights to the extra $500 for each of the three
weekends worked.
ISSUE: The legal issue relates to consideration and promissory estoppel.
RULE: Under Stilk v Myrick, if a promisee does no more than perform their existing contractual
duties, they are not providing sufficient consideration for a promise of extra payment

APPLY: For Katie to have a contractual right to the extra money, she must show that she is doing
something more than what her existing employment contract requires. However, Katie worked on
weekends. This appears to be more than what her employment contract requires.

Katie may also possibly have a claim in promissory estoppel which operates as an
exception to the requirement for consideration (Waltons Stores v Maher)

CONCLUSION: Katie may be successful in establishing promissory estoppel as a cause of


action.

11. You are a company director of a company called Finance Solutions Pty
Ltd. You need to lease office premises to run the company services from. You
approach a landlord of existing premises which premises you think would be
perfect to run your company from. The landlord, James Fitzgerald, tells you the
premises are currently leased to business tenants but that the lease to the tenants
will expire in two months and that he would then be happy to enter into a fixed
term five year lease contract with you. You shake hands with James and begin
to organize for the transfer of your company business from its old address to the
new premises.
You contact the Australian Securities Investment Commission and execute
necessary documentation. You contact all the company's customer base and
advise them of the company's new address. You also arrange for and pay an
architect in advance who is going to refurbish the interior of the new premises to
suit your company's business look.
Meanwhile James Fitzgerald is negotiating with his current tenants for them
to extend their current lease at an increased price for a further ten years. He
knows they are considering paying a higher rent and he is waiting for them to
make up their mind before he commits to you. He delays signing any contract
with you telling you "everything is fine, we will sign shortly".
It transpires that James actually re-leases the premises to his current
tenants instead of entering into a lease contract with you.
Do you have any legal rights against James Fitzgerald?

ISSUE: The legal issue in this matter is whether you can rely on the doctrine of promissory
estoppel to enforce your agreement with James Fitzgerald.

RULE: Promissory estoppel prevents a promisor from reneging on promises that they have made
where it would be unconscionable or unfair for them to do so: Waltons Stores v Maher.

3 elements need to be satisfied.


Downloaded by ahmad as (ksa-m29@hotmail.com)
lOMoARcPSD|3248894

20

APPLY: First, James intended you to rely upon his promise that you could lease the premises for
a fixed term of 5 years. Second, you have in fact relied upon that promise, and if that promise is
not kept, you will suffer material damage. Here you have relied upon the promise as you have filled
in and paid for the appropriate changes with ASIC, contacted your client base informing them of
the changes, and have contacted and paid for an architect to refurbish the interior. Finally, it would
be deemed unconscionable (unfair) for James to break his promise to you.

CONCLUSION: For the above reasons it could be deemed you have a valid contract with James
due to the doctrine of promissory estoppel.

12. Aaron has for many years worked on his parent's farm as a partner in the
family business. Aaron is an experienced house builder but decided to give up his
successful business to help his elderly parents on the farm for no payment.
Several months ago, Aaron decided to marry and was keen to return to his
business to better provide for his family. Knowing this, Aaron's parents promised
him that if he continues to work on the farm they will divide-off and give to him part
of the farm property so that he could build his own home.
In reliance on this promise, Aaron continued to work on the farm and
obtained a large bank loan to pay for the construction of his new home. The
parents promised Aaron they were arranging with their lawyer to execute a
contract to evidence this transfer of the property. However after many months and
with the house almost completed, no formal contract had yet been entered into.
Unknown to Aaron, the parents were reluctant to proceed with this
arrangement. They wanted to give the entire farm to their daughter, Marion.
The parents told their lawyer to "go slow with the preparation of the contract".
Aaron has recently spoken to his parent's lawyer and was told that his
parents now refuse to sign any contract and want him to leave the farm
immediately. Advise Aaron as to any relevant common law rights he has against
his parents.

ISSUE: The legal issue concerns promissory estoppel.


RULE: 3 elements:
1. The promisor intended the promisee to rely upon a clear and unambiguous
promise.
2. The promisee has, in fact, relied upon the promise by changing their circumstances,
and if the promisor does not keep their promise, the promise will suffer a material
disadvantage.
3. It would be unconscionable (unfair) for the promisor to break their promise.
Note also difference between using it as a shield (Central London Property Trust Ltd v
High Trees House Ltd) or as a sword (Waltons Stores v Maher)

APPLY:
CONCLUSION: Based on the preceding discussion it is likely that Aaron will be successful in
enforcing the promise.

Downloaded by ahmad as (ksa-m29@hotmail.com)


lOMoARcPSD|3248894

21

Week 8 (ch5 and ch6)


(a) Making Deals - Negating a Contract
(b) Terms and Non-contractual Representations

1. Huang, a 17 year old, bought a Pierre Cardin suit from Charles for $300 on ebay.
Huang wanted a suit so that she could go to an interview for a job as an intern at a
market research company. When Charles sent Huang an account for the suit Huang
refused to pay.

Advise Charles whether he can enforce the contract.


ISSUE: The legal issue concerns contracting with minors

RULE:
• Under s 3 of the Age of Majority Act 1977 (Vic), a person under 18 is a minor.
• Contracts for necessaries are enforceable against a minor, but must be in accordance with the
minor’s station in life. Necessaries includes food, clothing, accommodation and education.
• It is a question of law of what is a necessary. Goods Act 1958 (Vic) s 7: “necessaries are suitable to
condition of life of minor and to his requirements”
APPLY: Here, Huang is 17 years old and is therefore a minor.

The facts tell us she needs a suit for a job interview.

As long as Huang does not have another suit suitable for a job interview and the suit she bought is in
keeping with her situation in life, the contract is enforceable against her.

CONCLUSION: The contract is enforceable against Huang.

2. What does it mean not to consent to a contract? Identify and describe the different
grounds or legal bases which a person may argue to have a contract set aside or
made unenforceable on the basis of lack of consent.
• MISTAKE
• 3 categories:
• Unilateral: A one sided mistake, made by only one of the parties (Taylor v Johnson (1983))
• Common: Where both of the parties are mistaken about a fundamental aspect of the
contract (Leaf v International Galleries [1950])
• Mutual: Where both parties believe they have reached an agreement but are actually
thinking different things (Raffles v Wichelhaus (1864))
• DURESS
• A threat of harm made to another person to pressure them to enter into a contract:
Barton v Armstrong [1973].
• UNDUE INFLUENCE
• Unfair influence by one party to a contract over the other party, making the contract
voidable by the other party: Allcard v Skinner (1887); Johnson v Buttress (1936).
• UNCONSCIONABILITY
• Unfairly taking advantage of another person’s special weakness or disadvantage.
• A special weakness or disadvantage can include inability to speak or read English,
illiteracy or lack of education
• Commercial Bank of Australia v Amadio;

3. Adam entered the Reality Art Gallery and saw a painting which he immediately
liked. On making enquiries to Rudy, the owner of the gallery, he was told the
particular painting was a Vermeer landscape and had a price of $500,000. Adam
saw this as a good investment and so he purchased the painting. Some months
later a scandal broke out involving a master forger who specialised in Vermeer
paintings. Adam became very worried and after seeking an expert evaluation, his
worst fears were confirmed. He further learnt that the painting has a value of about
$1,000.
Adam wishes to know whether he has any legal remedies. Advise him.

Downloaded by ahmad as (ksa-m29@hotmail.com)


lOMoARcPSD|3248894

22
ISSUE: The legal issue in this question concerns mistake and misrepresentation

RULE:
• Common Mistake: Where both of the parties are mistaken about a fundamental aspect of the
contract (Leaf v International Galleries [1950])
• Misrepresentation: based on false statement of fact; addressed to person who is misled; intended
to induce the contract.

APPLY:
• Mistake as to quality: (Leaf v International Galleries) where it was held that, although the parties
were mistaken about the identity of the painter, the contract was not conditional upon the truth of this
belief.
• Misrepresentation: Discuss whether it is innocent, negligent or fraudulent.

CONCLUSION: Adam may not be successful in obtaining a remedy for common mistake.

However he may rescind the contract for innocent misrepresentation, or if he is able to prove negligent or
fraudulent misrepresentation, terminate the contract and seek damages.

4. Changying recently bought her own cafe. However business has been slow ever
since she opened the cafe and bills can only be paid out of an overdraft facility
provided by the Eastern Suburbs Bank. The bank however has now advised her
that it can no longer extend credit without some form of security. Changying's
boyfriend, Sonny, owns his own house. Sonny is very much in love with Changying
and wishes to please her. While Sonny is recuperating in hospital from concussion
sustained in a football match, Changying and the bank's loans officer get Paul to
sign a contract of guarantee to secure Changying's business debts using his house
as security. When signing, Sonny understands the nature of the document but not
the extent of his potential liability. He also has no idea that Changying's business
has not been breaking even. Changying's business goes broke and the Eastern
Suburbs Bank seek to recover $100,000 from Sonny under the contract of
guarantee.

Advise Sonny as to his legal position.


ISSUE: The legal issue in this matter is whether Sonny can set aside the contract on the grounds of
unconscionable conduct.

RULE:
• Unconscionable conduct involves unfairly taking advantage of another person’s special weakness or
disadvantage.
• The following requirements must be met:
1. One party has a special weakness or disadvantage.
2. The other party knows of this weakness or disadvantage.
3. The other party uses or takes advantage of the weakness or disadvantage.
(Commercial Bank of Australia v Amadio (1983)

APPLY: To have the contract set aside, the first matter Sonny must establish is that he has a special
weakness or disadvantage. He could argue that he is very much in love with Changying and wished to
please her, coupled with the fact that he was recuperating from concussion in hospital. Thus there is a
special weakness, as in Louth v Diprose (1992). It is arguable Changying knew of this weakness and
exploited it to her advantage. Bank also did not take steps to ensure he understood documents.

CONCLUSION: If Sonny is successful in establishing unconscionable conduct, the guarantee will be void.

Downloaded by ahmad as (ksa-m29@hotmail.com)


lOMoARcPSD|3248894

23
5. Phillips is the chairperson of the board of a large public company called STE Ltd.
Phillips has worked closely with the Managing Director of the company, Jackson, for
many years but recently their relationship has deteriorated. Jackson is very keen
that Phillips executes certain company contracts that will see Jackson make
enormous profit. Phillips believes that to sign such contracts will be a breach of his
fiduciary duties to the company and does not wish to sign. However, Jackson is
very intimidating and says to Phillips "I will see to it that you never work for this
company or any other company again". Phillips is aware of rumours that Jackson
has in the past 'ended' the careers of a few other company officers. Accordingly
Phillips signs the contracts with Jackson standing menacingly over him.

Does Phillips have any common law grounds pursuant to which he could arguably
terminate the contracts once signed?
ISSUE: The legal issue in this matter is whether the contract is voidable by Phillips on the grounds of
economic duress.

RULE: Economic duress involves a threat being made to another person’s economic or financial wellbeing to
pressure them to enter into a contract.

The following requirements must be met:


1. One of the parties has expressly or impliedly threatened the other party with (financial) harm;
2. The threat of the (financial) harm contributed to the threatened party’s decision whether or not to
enter into the contract.
APPLY: The facts show that Jackson has stated to Phillips that if Phillips does not sign the contracts for
Jackson, Jackson will see that he never works for this or any other company again. Phillips is aware that
Jackson has ended the careers of several other officers. In addition Jackson stands over Phillips whilst he
signs the contracts.

Arguably, this is a form of economic pressure that can render the contract voidable: North Ocean Shipping
Co Ltd v Hyundai Construction Co Ltd [1979] QB 705

CONCLUSION: Based on the preceding analysis, it is likely the contract Phillips entered is voidable.

7. What is the difference between a contract which is rendered illegal and


unenforceable because its performance is illegal versus illegal conduct which is
incidental to the performance of the contract?
• A contract which is rendered illegal and unenforceable because its performance is illegal
refers to contracts for the commission of a crime or tort, to evade paying taxes or prevent
the administration of justice: Public Service Employees Credit Union Cooperative Ltd v
Campion (1984). This means that the very nature of the contract is illegal. For example, two
people agreeing that one of them will commit a murder in exchange for money.
• A contract which involves illegal conduct incidental to the performance of the contract
means that the actual contract is not illegal but some aspects of performing it are:
Fitzgerald v FJ Leonhardt Pty Ltd (1997) 189 CLR 215. For instance, a contract for
transportation of goods such as food or clothing is perfectly legal, however if a truck driver
speeds whilst transporting goods, he may receive a fine or lose demerit points.

8. You are a business solutions adviser advising clients on choosing and


establishing business structure solutions. A new client engages you to incorporate
and register a new company for them. You set up the company in accordance with
the Corporations Act 2001 (Cth) but in contravention of this legislation forget to
obtain the written consent of the Managing Director (your client). When requested
to do so by the Australian Securities and Investments Commission (ASIC) in order
to process the registration form, you are too embarrassed to ask your client and so
forge a written consent yourself. Subsequently, incorporation occurs upon
registration of the new company. As agreed, you invoice your client, but upon
discovering that you forged her written consent she refuses to pay for your
services arguing that you acted illegally.

Is the contract between your client and you legal and enforceable?
Downloaded by ahmad as (ksa-m29@hotmail.com)
lOMoARcPSD|3248894

24

ISSUE: The legal issue in this matter concerns legality of contracts

RULE: A contract may be for a legal act, however, if an illegal act occurs whilst performing the contract, the
contract may be deemed invalid.

In Anderson Ltd v Daniel [1924] 1 KB 138 a contract for sale of fertiliser was deemed illegal and unenforceable
because it contravened legislation.
In Fitzgerald v FJ Leonhardt Pty Ltd (1997) 189 CLR 215 a contract to drill boreholes on land was deemed
valid and enforceable despite the borehole driller not holding a permit required by legislation.

APPLY: On the facts we are unsure as to the wording of the Corporations Act 2001 (Cth), however the
contravention appears to be quite serious because it involves fraud. The form will need to completed again,
this time with the real signature of your client. It is therefore likely that the court would follow Anderson Ltd v
Daniel [1924] 1 KB 138 and hold that the contract is unenforceable.

CONCLUSION: Based on the preceding analysis, it is likely the contract will be deemed unenforceable.

9. What is the difference between a statement made before a contract is entered into
(a mere representation made prior to the contract) versus a term of a contract?
When might a pre-contractual statement be a term of a contract?
 A mere representation is a statement likely said during negotiations, but it does not necessarily
follow that it will become part of the contract. Mere representations have no contractual significance
(unless they are materially misleading) and confer no rights on the party to whom they are
addressed.
 A term of a contract can be express (explicitly agreed to by the parties either verbally or orally) or
implied by conduct or through previous dealings. A term of a contract forms part of the contract and
outlines the parties’ rights and obligations.
 A pre-contractual statement may become a term of the contract if the parties intended for it to
become an enforceable obligation: Pacific Carriers Ltd v BNP Paribus (2004).
 The Court may also decide that the pre-contractual statement is a ‘collateral contract’ meaning the
statement was intended to have contractual significance and one of the parties would not have
entered into the main contract without it: De Lassalle v Guildford [1901] – promise re drains was CC
to enter into lease

10. Brett and Danielle negotiated for the sale of Danielle's accounting practice. During
negotiations, Danielle stated that the gross takings of the business were $3,000 per
week. Soon after these negotiations, extensive road works commenced outside the
business premises. Consequently access to the practice was restricted and no new
clientele walked in off the street. Additionally, existing clients were frustrated by the
lack of car parking and some clients took their business elsewhere. Gross takings
for the business dropped to $1,500 per week. Two months later, Brett purchased
the business and found the takings significantly less than Danielle had represented.

Advise Brett of his common law rights.


ISSUE: The legal issue in this question concerns misrepresentation

RULE: There are 4 elements that must be satisfied to prove misrepresentation:


1. representation was made which is false
2. representation is one of fact
3. representation is addressed to party misled
4. representation was intended to induce and did actually induce the contract.

APPLY:
• Can silence be misrepresentation? Normally no, but if statement becomes untrue, seller has obligation
to correct: With v O’Flanagan
• Is the misrepresentation innocent, negligent or fraudulent?
• Is the statement a term of the contract or mere representation?

CONCLUSION: Danielle’s silence on the change of circumstances (the commencement of extensive road
works outside the store) would amount to a misrepresentation under With v O’Flanagan. Brett may therefore
rescind the contract and claim damages.
Downloaded by ahmad as (ksa-m29@hotmail.com)
lOMoARcPSD|3248894

25
10 Generally if a contractual term is in a written contract that has been signed by the
parties, it is a binding and enforceable term of the contract. Is this the case even if
one of the parties who signed the contract has not actually read the written contract?
• Yes this statement is correct. In L’Estrange v F Graucob Ltd [1934] the court decided L’Estrange was
bound by the contract despite the fact he had not read it.
• However, if one of the parties was unaware of the contract contents because of a disability or
disadvantage, the court may find the contract is void and unenforceable: Petelin v Cullen (1975) 132
CLR 355. This is known as the principle of non est factum (it is not my deed).

11 Paul Fernando owns a web design business which operates as a business in


Melbourne. Paul has not completed his taxation returns for the business for the last
five years. The Australian Tax Office (ATO) has issued Paul with a notice requesting
all outstanding taxation returns be submitted by 30 June. Paul hires Graeme Smith
to do the work. Paul and Graeme agree that for a $2000 fee Graeme will prepare all
the necessary documentation and submit it on behalf of Paul to the ATO by 30 June.

Paul has been studying Commercial Law and drafts a contract of engagement
which includes the following terms:
Term One: For $2000 Graeme Smith will prepare all ATO necessary documentation
on behalf of Paul Fernando for his web design fitness club business for
the last five years and submit this documentation by 30 June.
Term Two: Graeme Smith takes no responsibility for any error or miscalculation
or other problems with the ATO during and after the provision of
services to Paul Fernando.

Paul gives Graeme the contract of engagement but as Graeme trusts Paul he does
not feel he needs to read the contract.

There is significant documentation to attend to and Graeme works hard. However,


because of personal concerns Graeme is preoccupied and forgets to lodge the
necessary documentation with the ATO before the 30 June deadline.

On 1 August the ATO fines Paul $2000 for not submitting the requested taxation
returns.

Critically evaluate, with reference to case-law, whether Term Two in the purported
contract would exclude Graeme being liable for breach of contract for being unable
to submit the taxation documentation by 30 June.
ISSUE: The legal issue in this matter concerns disclaimers.

RULE: A disclaimer is a statement that one of the parties will not be in breach despite failing to perform one or
more of their contractual obligations. Disclaimers are also known as ‘exclusion clauses’.

Whether a disclaimer will protect a person from liability for breach of contract depends on:
1. Whether the disclaimer is in fact a term of the contract;
2. Whether the disclaimer will be interpreted as applying to the particular breach.

APPLY:
1. Given the disclaimer is contained in a documental that appears to be contractual in nature (a contract
of engagement) it is likely a court would find it is in fact a term of the contract.
2. On the facts the statement ‘errors, miscalculations or any other problem’ may be deemed too vague.
Does it extend to failing to lodge the tax return by the due date? If the court decides it is too vague,
Graeme will not be allowed to rely on that clause.

CONCLUSION: Based on the preceding analysis, it is likely that Paul may be entitled to damages of $2,000
for the fine imposed by the ATO and the $2,000 he paid to Graeme to complete the work.

Downloaded by ahmad as (ksa-m29@hotmail.com)


lOMoARcPSD|3248894

26
12. Anastasia stopped her car at the barrier entrance to a car park and an automatic
ticket issuing machine projected a ticket which Anastasia took in accordance with a
large notice attached to the barrier which read:

"Parking at Owner's Risk - Take Ticket from Machine. Pay when leaving."

Anastasia put the ticket in her pocket without looking at it, the barrier raised and
Anastasia drove through and parked her car. On returning to pick up her car she
found it completely demolished beneath a large concrete pipe which had fallen from
a crane owned by the parking station proprietors who were working on an adjacent
drainage ditch. The parking ticket had the following notice printed on it:

"It is a condition of the issue of this ticket that vehicles are parked on these
premises at the owner's risk and responsibility. The proprietors accept no
responsibility for loss of damage to vehicles in the parking area whether
caused by negligence or in any manner whatsoever".

Advise Anastasia of her common law rights in relation to the notices on the barrier
and ticket.
ISSUE: The legal issue in this matter concerns disclaimers

RULE: To determine whether the exclusion clauses (also known as disclaimers) protect the car park
proprietors, the following issues need to be addressed:

• Incorporation: do the exclusion clauses form part of the contract? and, if so;
• Interpretation: do they cover the breach?
(Thornton v Shoe Lane Parking).

APPLY:

First exclusion clause has sufficient notice. Second exclusion clause not part of the contract.

First exclusion clause may be ambiguous. Second exclusion clause on the ticket is probably effective.

CONCLUSION: Exclusion clauses may or may not be effective depending on factual circumstances and
analysis.

Downloaded by ahmad as (ksa-m29@hotmail.com)


lOMoARcPSD|3248894

27
Week 9 (CH6)
Enforcing Deals - Remedies and Ending the Contract
1. Identify and describe to next year's Commercial Law students the different ways a
contract can come to an end.
1) Discharge by performance (everyone performs their obligations)
2) Discharge by agreement (parties agree to vary contract)
3) Discharge by frustration (unanticipated event occurs that alters agreement)
4) Discharge by breach (one party fails to perform – contract terminated)
5) Discharge by operation of law (law forces rights to merge e.g. where tenant buys house and becomes
owner)

2. Lucy is in her final year of her accounting degree and is looking for a job. She sees
advertised in the newspaper a position with a small accounting practice in Melbourne
which reads:

"Wanted: A qualified early career accountant. Please telephone David 9600


5222 to discuss the beginning of your career".

Lucy telephones David to discuss the advertisement. David tells Lucy that the job is
for an accountant graduate and advises that the job is fulltime and pays $3500 a
month. David also tells Lucy she always needs to arrive at the office by a.30am.
Lucy does not tell David she has not yet graduated from her accounting degree.

Lucy commences work and is very capable. After 29 days of arriving to work
punctually, Lucy arrives on a Monday at 10am. David becomes very angry and
refuses to pay Lucy her monthly salary. He tells her, "I owe you nothing. You are too
late for work and I have known for a while that you lied about being an accountant.
You are fired".

Critically evaluate, using case law only, the different legal grounds (if any) David
may argue which entitle him to fire Lucy (or in other words terminate the contract (if
any) with her. Note this question also involves issues relating to week a
consent/misrepresentation.

ISSUE:
• The legal issues in this matter involve discharge by breach – condition or warranty, substantial
performance and misrepresentation.
RULE:
• A condition is a term of the contract with fundamental importance. A breach of a condition
entitles the aggrieved party to terminate the contract and seek damages: Poussard v Spiers &
Pond (1876).
• A warranty is a term of the contract with lesser importance. A breach of a warranty entitles the
aggrieved party to damages only: Bettini v Gye (1876).
APPLY:
• In general, without the parties emphasising in a contract that ‘time is of the essence’, punctuality
is likely to be a mere representation or a warranty.
• David would argue arriving on time is a condition of the contract.
• Lucy would counter argue that she has substantially performed.
• David would also argue Lucy has misrepresented her qualifications. If he can prove fraudulent
misrepresentation, he can terminate the contract and recover damages.
CONCLUSION:
• Lucy is entitled to be paid for the work performed to date.
• However, David has a strong argument that Lucy has knowingly misrepresented herself as an
accounting graduate, when in fact, she is still a student in her final year. It is therefore likely
David will be permitted to terminate the contract.

Downloaded by ahmad as (ksa-m29@hotmail.com)


lOMoARcPSD|3248894

28

3. Packer University wishes to appoint a new Vice-Chancellor. It finally offers the job to
Professor Chris Baker, of New York University. A contract is signed on 1 January,
2015, providing for a term of five years with a package worth $100,000 a year.
Professor Baker is to take up the position on 1 June, 2015.

On 1 May 2015, Professor Baker is seriously injured in a skiing accident, and on


medical evidence it is clear that she will not be able to take up the appointment with
Parker University until January 2016.

The University wishes to know whether or not it can terminate Professor Baker's
contract.
ISSUE:
• The legal issue in this matter concerns discharge by breach and discharge by frustration.
RULE:
• A term of a contract can be a condition or warranty (Associated Newspapers v Bancks; Bettini v
Guy). Unless contract states time is of the essence, complying on time will usually be a warranty.
• If a contract becomes impossible to perform due to an unanticipated event, it may be frustrated:
Taylor v Caldwell
APPLY:
• The Professor’s lateness is likely to be regarded as a breach of warranty, entitling the University to
damages only.
• The skiing accident may be regarded as a frustrating event. There are 4 elements to be satisfied.
CONCLUSION:
• Based on the preceding analysis, it is likely that the skiing accident is a frustrating event causing the
contract to be void. The University can cease the Professor’s employment without paying her.

4. Riley Constructions Pty Ltd enters into a fixed price contract of $500,000 with Allan
for the construction of a block of office premises. When the offices have been built to
window height, Riley Constructions Pty Ltd finds that it will be uneconomic to continue.
Can Riley Constructions Pty Ltd legally obtain payment for work done and the
materials used thus far?

Would your answer be different if Riley Constructions Pty Ltd had built the whole
block of office premises but had failed to put tiles in the staff kitchenettes?
ISSUE:
The legal issue in this matter is discharge by performance.
RULE:
The general rule is that for a contract to be discharge by performance, exact performance is
necessary. However there are some exceptions to the rule:
1. Divisibility of contract
2. Partial performance has been accepted
3. Substantial performance
APPLY:
• Riley Constructions has not complied with its obligations and none of the exceptions apply. Building
offices to window height is not acceptable because none of the buildings are complete. Builder is not
entitled to be paid: Cutter v Powell
• Where the whole block is built except for installation of the tiles in the staff kitchenettes, the situation
is entirely different since there has been substantial performance. Here the builder is entitled to full
payment less a sum to represent the cost of tiling the kitchenettes. See Hoenig v Isaacs.
CONCLUSION:
Based on the preceding analysis, the builder is not entitled to be paid. If all of the work was
completed, the builder could receive payment less the cost of the tiling.

5. One way a contract can come to an end is via the doctrine of frustration. What is
this doctrine and what needs to be established for a contract to be frustrated?

The doctrine of frustration provides that a contract may be terminated due to unanticipated impossibility or
extreme difficulty of performance.
A contract will be frustrated if all of the following requirements are satisfied:
1. A ‘supervening event’ has made performance of the contract either completely impossible or at least
impossible to perform in the way originally envisaged by the parties.
Downloaded by ahmad as (ksa-m29@hotmail.com)
lOMoARcPSD|3248894

29
2. Neither party caused the supervening event.
3. The contract did not provide for the supervening event either expressly or by implication.
4. It would be unjust to compel either party to proceed with the contract.
For instance in Taylor v Caldwell (1863) a contract for the hire of a hall was frustrated because the hall was
destroyed by fire.
These elements are important because you must establish each one to prove frustration!

6. You have leased business premises to run your boutique information technology
company from. The lease is to commence 1 July 2015. Just before 1 July 2015 the
business premises you have leased are destroyed by fire. Consequently the lessor
advises she cannot lease the premises to you. You are unable to find suitable
premises by the 1 July 2014 and have nowhere to operate your company from.

Are you entitled to damages for breach of contract?


ISSUE:
• The legal issue in this matter concerns termination of a contract through frustration.
RULE:
• A contract will be frustrated if four requirements are satisfied:
1. A ‘supervening event’ has made performance of the contract either completely impossible or at least
impossible to perform in the way originally envisaged by the parties.
2. Neither party caused the supervening event.
3. The contract did not provide for the supervening event either expressly or by implication.
4. It would be unjust to compel either party to proceed with the contract. Taylor v Caldwell (1863)
APPLY:
• The first element is satisfied because the fire constitutes a ‘supervening event’ that has now made the
contract impossible to perform. In other words, you cannot have a lease for a property if the property
no longer exists.
• The second element is satisfied because on the facts neither party appears to have caused the fire.
• The third element appears to be satisfied because the facts do not refer to an express clause (or any
implied conduct) which addresses the parties’ position in the event a fire destroys the premises.
• Finally, it would be unjust to compel you to proceed with a contract for the lease of premises that are
no longer in existence.
• The facts are similar to Taylor v Caldwell (1863) where a contract for the hire of a hall was frustrated
because the hall was destroyed by fire.
CONCLUSION:
• Based on the preceding analysis, you are not entitled to damages for breach of contract because the
contract has been frustrated.

7. Generally a third person who is not a party to a contract cannot enforce the contract
(nor have it enforced against them). What is the doctrine called in law and can you
think of a hypothetical to apply this principle to.
 This statement refers to the doctrine of privity, a principle that only a party to the contract is legally
obliged to perform a contract, and has the right to enforce a contract.
 For example, if you enter into a contract with John’s Garden Services to have your grandmother’s
lawn mowed, the contract is between you and John’s Garden Services. You provide consideration
(say, $100) in return for a service (lawn mowing). In the event John’s Garden Services only mow half
of the lawn, you can sue for breach of contract (as opposed to your grandmother). This is because
your grandmother is not a party to the contract (she is simply getting the benefit of it).

8. Identify and describe the different types of remedies that may be available for
breach of contract.
 RESCISSION: This involves the cancellation of a contract (as a consequence of a breach) and the
restoration of the parties to their original position. For example, John and James enter into a
contract for the sale of a computer, and John breaches the contract. Following a rescission order,
James must return the computer to John, and James must repay the purchase price to John.
 DAMAGES: This involves monetary compensation being paid to restore the plaintiff to the position
they would have been in if the breach or misrepresentation had not occurred. For instance, if John
defaults on a loan contract with you, you are entitled to damages to the total amount of repayments.
See Hadley v Baxendale (1854) regarding damages.
 EQUITABLE REMEDIES:
– SPECIFIC PERFORMANCE: This is a court order directing a party to fulfil their contractual
obligation: Dougan v Ley (1946)
– INJUNCTION: This is an order forbidding someone from engaging in a particular conduct
Downloaded by ahmad as (ksa-m29@hotmail.com)
lOMoARcPSD|3248894

30
that will be in breach of the law or infringe on the legal rights of another: Buckenara v
Hawthorn Football Club [1988]
 STATUTORY REMEDIES:
– Goods Act 1958 (Vic) ss 44-55 outlines seller’s remedies.
– Goods Act 1958 (Vic) ss 57-59 outlines buyer’s remedies.
– Australian Consumer Law (referred to in later topics).

9. Bells Surfboards Pty Ltd of Victoria, engages a world famous board rider, Justin
Stevens, for a series of surf riding exhibitions in April, using Bells surf boards. Justin
and Bells Surfboards agree that a commercial film "Endless Summer" will be made
of the exhibitions. Two days before the exhibitions were to be held Justin informs
Bells Surfboards that he is going to move to Bali to surf professionally there.
Consequently the exhibitions have to be cancelled.

Bells Surfboards sue Justin for breach of contract, claiming damages for:
 Loss of profits because of a reduced sale of surfboards arising because of the
cancellation;
 Promotional expenses;
 Loss of profits on the projected film "Endless Summer" and
 Loss of profits on a radio feature which Bells Surfboards had negotiated with 6DA, a
radio station.

Which of the above will likely to be assessed for damages, and why?

ISSUE:
• The legal issue in this question is remoteness of damage.
RULE:
• The case of Hadley v Baxendale established a two part test, that damages may only be claimed if:
1. Where they naturally arise from the breach in the usual course of things; and
2. It was reasonably in contemplation of the parties at the time of contracting.
APPLY:
• The loss of profits can be recovered because they fall under the first limb
• The promotional expenses can be recovered because they fall under the first limb
• The film profits can be recovered because they were in contemplation by the parties
• The radio feature is unlikely to succeed.
CONCLUSION:
• Based on the preceding analysis, it is likely that the first 3 items can be compensated for because
they fall within the two limbs of the remoteness of damage test.

10. Wei Lin runs a successful marketing firm. Her marketing firm practises a highly
specialised area marketing which requires her staff to undergo expensive training
and acquire specialised skills to carry out their employment duties. Wei Lin is
looking to expand her business and employs Raymond. Raymond signs the
following employment contract which includes the following terms (Clauses).
Clause 1.6 - In the event of resignation by the employee, the employee is to
give three months of notice to the employer.
Clause 1.7 - The employee upon termination of employment cannot work for
any other law firm providing law advice or services within Australia for
five years.
During his two years in Wei Lin's employment Raymond acquires highly confidential
information about the firm's clients and, because of the extensive training he is
given, knowledge in highly technical aspects of international marketing practices.
Over time, Raymond is given client matters of his own to provide advice to and
services for.
After two years of working for Wei Lin, Raymond applies and is offered a
position at a big events management firm, Management Events Marketing (MES).
MES provides advice and services in event planning and marketing. Raymond
gives Wei Lin one week's notice and leaves to work for MES. Wei Ling is short
Downloaded by ahmad as (ksa-m29@hotmail.com)
lOMoARcPSD|3248894

31
staffed and needs Raymond to finish working on his current clients' matters. Wei
Lin cannot find anyone to replace Raymond and wants him to return to work for
her. Additionally Wei Lin's business is losing $2000 a week as Raymond's clients
will not pay for services not provided to them. To make things worse, Raymond's
clients are beginning to move their business to MES to continue working with
Raymond.
Wei Lin wants to obtain remedies for breach of contract. Raymond counter-
argues that the contract of employment with Wei Lin is unenforceable on a number
of grounds.

(A) Critically evaluate, using case-law, on what different grounds (if any)
Raymond might argue that the contract of employment between himself and
Wei Lin is unenforceable?
AND
(B) If the contract of employment is enforceable, identify the most likely breaches
of contract by Raymond and identify and describe what the different
remedies are that Wei Lin may be legally entitled to.
ISSUE:
• The legal issue in this matter concerns restraint of trade clauses.
RULE:
• The courts will refuse to enforce a restraint of trade clause if it is unreasonable in terms of:
– Time;
– Geographical area; or
– Scope of business
In Forster & Sons Ltd v Suggett (1918) the court held that a restraint of 5 years on an employee from
engaging in glass or glass bottle manufacturing for 5 years was reasonable. Similarly, in Nordenfelt v
Maxim Nordenfelt Guns and Amunition Co Ltd [1894] the court held that a term prohibiting
Nordenfelt from starting a competing business anywhere in the world for 25 years was reasonable.
APPLY:
• Pursuant to the Nordenfelt principle, restraint of trade clauses are void and unenforceable unless
they are reasonable by reference to the interests of the parties and the public. The restraint must
also be reasonable in terms of time and space.
• The facts state that the employees undergo expensive training and acquire specialised skills. In
particular, Raymond has acquired skills in highly technical aspects of international marketing
practices.
• Wei Lin cannot restrict Raymond using general skills and knowledge acquired at work, but may be
able to restrict Raymond’s use of confidential information acquired during his employment: Forster &
Sons Ltd v Suggett (1918). The restraint must be reasonable. Here the restraint is for a 5 year period
which does not appear to be unreasonable given the expensive training and specialised skills
Raymond has acquired during the course of his employment.
CONCLUSION:
• Raymond will be able to argue that the restraint of trade clause is unreasonable but it is unlikely that
his argument would be successful.

If the contract is enforceable, what are the likely breaches and remedies?
• If clause 1.6 is deemed to be a condition of the contract, it will entitle Wei Lin to terminate the contract
and seek damages. If clause 1.6 is deemed to be a warranty, it will entitle Wei Lin to damages only.
• If Wei Lin seeks damages she will need to satisfy the two limbs in Hadley v Baxendale
• An alternative to damages would be for Wei Lin to seek an order for specific performance, to force
Raymond to fulfil his contractual obligations.
• An additional alternative would be for Wei Lin to seek an injunction, to prohibit Raymond from
engaging in conduct that would amount to a breach of contract.

11. In what cases will the courts grant specific performance of a contract?
A court will grant the equitable remedy of specific performance when an order for damages would not be
adequate, eg where the subject-matter of the contract is unique or rare (such as antiques or art work) or
where it has special personal value to the claimant (such as sentimental reasons). However, a court
will not grant specific performance if the contract is for personal services.

Downloaded by ahmad as (ksa-m29@hotmail.com)


lOMoARcPSD|3248894

32
Week 10 (w/c May 11) CH 8
Dealing with Consumers - General Protections
1. Why do consumers deserve extra legal protection?
 Consumers are at a disadvantage when dealing with a business
 Business knows more about their products than the consumer
 Business has more business experience and greater access to resources
 Traditional rules of contract law are generally based on ‘caveat emptor’ and therefore
offer little assistance to consumers.
 There is no real equality of bargaining power between business and consumer
 So, legislation seeks to protect consumers
2. How is a 'consumer' defined in the Australian Consumer Law ('ACL')?
Section 3 ACL provides that a person is a consumer if:
• The goods are less than $40,000; or
• If goods are more than $40,000, the goods or services is of a kind ordinarily used for person,
domestic or household use or consumption
• BUT the goods must NOT be for re-supply or to be used up or transformed in trade or
commerce.

3. What needs to be proved in an action for breach of ACL s 18 (prohibiting misleading


and deceptive conduct)? Page 373
Business has ‘engaged in conduct’: this includes acts, statements, promises, the failure to do or
say something. It also includes silence (Henjo Investments v Collins Marrickville)
The conduct was ‘in trade or commerce’. The conduct must therefore be in a commercial context,
and does not cover political or academic statements
The conduct was ‘misleading or deceptive or likely to mislead or deceive’. For this it is
necessary to use the objective test which is done by reference to the target audience. Statements
that are literally true may be misleading. Exaggerations may or may not be misleading or deceptive
depending on the context

4. Can ACL s 18 be enforced by persons other than consumers?


Apart from consumers, interested members of the public and other businesses, including
competitors, may seek to enforce ACL s 18

See case of Gillette Australia Pty Ltd v Energizer Australia Pty Ltd (2002) 193
ALR 629 on page 371
5. What are the consequences of contravening ACL s 18?
There are no criminal penalties
However, the ACCC may issue notices or warnings
Consumers or persons harmed can take legal action and seek remedies, including:
- pecuniary penalties: ss 224-231
- injunction: ss 232-235
- damages: s 236
- compensation order for injured persons: ss 237-241
- declaration that contract is void, order to refund, repair, compensate, etc.

6. What is prohibited by ss 20 and 21 of the ACL?


According to s 22 of the ACL what types of matters may a court consider in
deciding whether or not a business has contravened s 21? Page 378. M ention 2
or 3 from 22...not all. Need to be relevant

Section 20 of the ACL prohibits unconscionable conduct generally in trade or commerce.


Section 21 of the ACL prohibits unconscionable conduct in trade or commerce, in connection with the supply
or acquisition of goods or services. Section 21 does not provide a definition of unconscionable conduct and
therefore it is helpful to refer to section 22.

Downloaded by ahmad as (ksa-m29@hotmail.com)


lOMoARcPSD|3248894

33
7. In what circumstances will a term of a contract be unfair and therefore void under
ACL s 23?
• It is a ‘consumer contract’: defined in s 23(2) as one for the supply of goods, services or an interest
in land to an individual whose acquisition is wholly or predominantly for personal, household or
domestic use
• It is a ‘standard form contract’: the terms are not negotiated and presented on a ‘take- it-or-leave-it’
basis. See s 27(2) for factors a court is to take into account: eg, where one party has all or most of
the bargaining power, where the other party was required to accept or reject the terms of the
contract
• Term is ‘unfair’ under s 24(1) if
(a) it causes significant imbalance in parties’ rights/obligations,
(b) not needed to protect the legitimate interests of the business,
(c) would cause detriment to consumer.

9. Your grandmother lives alone in the countryside and you visit her once a month.
She speaks limited English and has poor hearing. She supports her simple living
costs through receipt of a low pension.

One Friday afternoon a travelling salesperson knocks on your grandmother's door


and is very persuasive in his sales pitch relating to a $1100 'deluxe' vacuum
cleaner. Your grandmother is often lonely in between visits from you and invites the
salesperson in for some tea and biscuits. The salesperson can clearly tell that your
grandmother understands little of what he is saying to her and also cannot hear him
very well. He can see from her house that she has wooden floorboards but still
continues talking about the wonders of the deluxe vacuum cleaner. Your
grandmother is persuaded to purchase the vacuum cleaner and gives the
salesperson her savings of $1100 which she keeps in the biscuit tin. Your
grandmother believes the salesperson will be back each week to talk with her and
to do the vacuuming.
It has been one month since your last visit and three weeks since the salesperson
sold the vacuum cleaner to your grandmother. When you visit her she is upset and
tells you she cannot afford to eat or pay her bills because she seems to have
purchased an expensive vacuum cleaner that she does not know what to do with.

You are a conscientious Commercial Law student and know your grandmother may
be able to take action against the salesperson under the Australian Consumer Law.
Identify and apply the relevant sections of the ACL to conclude whether or not your
grandmother would be successful in legal action taken against the salesperson?

• ISSUE: The legal issue in this matter is whether your grandmother would be successful in having the
contract for the vacuum cleaner set aside on the grounds of unconscionable conduct.
• RULE:
• Common law unconscionable conduct: CBA v Amadio
• Section 21 of the ACL prohibits unconscionable conduct in trade or commerce, in
connection with the supply or acquisition of goods or services.
• Section 22 of the ACL assists in determining whether a person has breached section 21 by
providing a list of matters a court may take into account in deciding whether a person has
behaved unconscionably.
• APPLY:
• A business will have contravened section 21 if the following requirements are met:
• 1. The business has ‘engaged in conduct’.
On the facts the travelling salesperson has engaged in conduct towards the grandmother.
• 2. The conduct was ‘in trade or commerce’.
The salesperson was selling vacuum cleaners and therefore the conduct was ‘in trade or
commerce’.
• 3. The conduct was in connection with the supply of goods or services to, or
acquisition of goods or services from, another person.
The conduct was in relation to the supply of a vacuum which is a good.
• 4. The other person was not a listed public company.
We would need to check if the company was publicly listed, however it is more likely that the
travelling salesperson was part of a smaller business.
• 5. The conduct was unconscionable.
Downloaded by ahmad as (ksa-m29@hotmail.com)
lOMoARcPSD|3248894

34
Your grandmother has a strong argument that the salesperson behaved unconscionably in
his dealings with her. Here, the salesperson was in a stronger bargaining position compared
to your grandmother, given she did not speak English very well: s 22(1)(a) ACL. The
salesperson used unfair sales tactics, by convincing your grandmother to buy a vacuum
cleaner even though she did not need it and that he would be back each week to clean the
floors: s 22(1)(d) ACL. It is also arguable your grandmother could have acquired a vacuum
cleaner for far less than $1,100: s 22(1)(e) ACL.
• The facts of this scenario are very similar to ACCC v Lux Distributors Pty Ltd [2013] where vacuum
salespeople called upon the homes of elderly people under the premise of conducting a free vacuum
cleaner maintenance check, but once inside the homes engaged in unfair and pressure sales
techniques to induce them to purchase a new vacuum cleaner. The court found that the conduct was
unconscionable, and that consumers must be dealt with honestly, fairly and without deception and
unfair pressure.
• CONCLUSION:
• Based on the preceding analysis, it is likely that your grandmother will be successful in having her
contract for the purchase of the vacuum set aside.

9. Telster, a new start up Australian telecommunications and technology business, are


keen to make lots of money. They employ a number of enthusiastic sales staff who
travel from door to door selling Telster smart phone handsets. Telster have
employed a Commercial Law graduate, Huang, to draft their standard form
contracts. The contracts are designed so that customers 'lease' the hand set for
$150 a month for 24 months. Huang is instructed to include a term (term 10) in the
standard form contract which provides:
"if a monthly payment is not made within 5 days of its due date, the customer
will incur a $700 late administration fee which fee will accrue by $20 a day for
each day the monthly payment and the late administration fee remain unpaid".
You have signed a contract to lease a new Telster hand set. After a months of
making timely payments you lose your job and cannot pay the July monthly $150
payment. After two weeks the Telster 'Debt Recovery' Team start phoning you
insisting you owe them $1130 which amount is increasing each day it remains unpaid.
Considering ACL s 23, how might you argue you should not have to pay this amount?

• ISSUE: The legal issue in this matter concerns unfair contract terms under the Australian Consumer Law.
• RULE: Section 23(1) of the ACL provides that a term of a consumer contract is void if:
(a) The term is unfair; and
(b) The contract is a standard form contract.
• This means that you have to satisfy three requirements:
1. The contract is a consumer contract.
2. The contract is a standard form contract.
3. The term is unfair.
• APPLY:
• The 1st requirement that must be satisfied is that the contract is a consumer contract. Section 23(3) ACL
defines that as a contract for the supply of goods or services used wholly or predominantly for personal,
domestic or household use or consumption. Here the contract is for a smart phone (a good) used
predominantly for personal use and therefore element one is satisfied.
• The 2nd requirement that must be satisfied is that the contract is a standard form contract. Section 27(2)
ACL outlines five factors that must be taken into account in determining whether a contract is a standard
form contract: the bargaining power of the parties, whether the contract was prepared by one party,
whether one party was required to accept or reject the contract as it was presented, whether another
party was given an opportunity to negotiate and whether the contract took into account specific
characteristics of another party to the transaction. Here, it appears the contract was presented as drafted
by Telster, with no room for the customers to negotiate or alter terms. Therefore the contract is a standard
form contract.
• The 3rd requirement that must be satisfied is that the term was unfair. Here you would argue that term 10
is an unfair term. To succeed you must refer to three requirements pursuant to s 24(1) ACL:
• the term causes a significant imbalance in the consumer’s rights as compared to the business’s rights
• the term is not reasonably necessary to protect the business’s interests
• the term causes any detriment to the consumer
• Term 10 would appear to cause a significant imbalance of power because it imposes severe and onerous
financial penalties on the customer for paying late (which the business does not itself face): Director of
Consumer Affairs Victoria v AAPT Limited [2006]. Term 10 is not reasonably necessary to protect Telster;
a smaller fine for paying late would be appropriate. Term 10 does cause a financial detriment to the
consumer because of the onerous financial penalty.
Downloaded by ahmad as (ksa-m29@hotmail.com)
lOMoARcPSD|3248894

35
• CONCLUSION: Accordingly, you may argue that term 10 is an unfair term and that it is void.

10. Telster have been now trading for a number of years and are now incorporated as a
company. Telster, advertises its new smart phone as "having coverage all over the
world". It in fact only has coverage in locations with adequate mobile network. On
what basis (if any) has Telster breached the Australian Consumer Law?
• ISSUE: The legal issue in this matter concerns misleading and deceptive conduct under the
Australian Consumer Law.
• RULE:
• Section 18(1) of the ACL provides that a person must not, in trade or commerce, engage in conduct
that is misleading or deceptive or is likely to mislead or deceive.
• A business will have contravened s 18 if all of the following requirements have been satisfied:
• 1. The business has ‘engaged in conduct’.
• 2. The conduct was ‘in trade or commerce’.
• 3. The conduct was ‘misleading or deceptive or … likely to mislead or deceive’.
• APPLY:
• The first requirement that must be satisfied is that Telster has ‘engaged in conduct’. Telster has
published an advertisement that makes a false claim, which constitutes ‘engaging in conduct’.
• The second requirement that must be satisfied is that the conduct was ‘in trade or commerce’.
Telster is an incorporated company publishing an advertisement, therefore the conduct is occurring
‘in trade or commerce’. Section 2 ACL defines trade or commerce as meaning trade or commerce
within Australia or between Australia and places outside of Australia. It includes any business or
professional activity
• The third requirement that must be satisfied is that the conduct was ‘misleading or deceptive or …
likely to mislead or deceive’. Given the advertisement claims Telster provides coverage all over the
world, when in fact it only provides coverage in locations with adequate mobile network, is
misleading or likely to mislead the public.
• Telster may argue the advertisement is simply ‘sales puff’ and that viewers would not take the claim
seriously. However, in Given v Pryor (1980) the court held an exaggeration constituted misleading
and deceptive conduct. If this case was followed, a court would likely find Telster’s advertisement
contravened s 18 of the ACL.
• CONCLUSION: Based on the preceding analysis, it is likely Telster has contravened s 18 of the ACL.

Downloaded by ahmad as (ksa-m29@hotmail.com)


lOMoARcPSD|3248894

36
Week 11 (w/c May 18)
Dealing with Consumers - Specific Protections

The following questions relate to the consumer guarantees under the Australian
Consumer Law ('ACL') discussed in Chapter 8 of the textbook Business Law.

1. Revisit the definition of 'consumer' under s 3 of the ACL. How would you define a
consumer for the purposes of this legislation applying to a consumer contract?

• Section 3(1) ACL provides that a person is taken to have acquired goods as consumer if the amount
paid for the goods does not exceed $40,000; or if the price of the goods exceeds $40,000, the goods
were of a kind ordinarily acquired for personal, domestic or household use or consumption; or the
goods consisted of a vehicle or trailer acquired for use principally in the transport of goods on public
roads.
• Section 23(3) ACL provides the definition of ‘consumer contract’ as a contract for the supply of
goods, services or an interest in land to an individual whose acquisition of the goods, services or
interests is wholly or predominantly for personal, domestic or household use or consumption.
• This means that the s 23(3) definition is narrower than the s 3 definition of consumer. This is because
the s 23(3) definition focuses on the buyer’s purpose in purchasing the product whereas the s 3
definition focuses on the type of product being purchased.

2. What is the definition of 'manufacturer' under s 7 of the ACL?


Section 7 (1) ACL provides that the term manufacturer includes the following:
(a) a person who grows, extracts, produces, processes or assembles goods;
(b) a person who holds himself or herself out to the public as the manufacturer of goods;
(c) a person who causes or permits the name of the person, a name by which the person carries on business
or a brand or mark of the person to be applied to goods supplied by the person;
(d) a person (the first person) who causes or permits another person, in connection with:
(i) the supply or possible supply of goods by that other person; or
(ii) the promotion by that other person by any means of the supply or use of goods;
to hold out the first person to the public as the manufacturer of the goods;
(e) a person who imports goods into Australia if:
(i) the person is not the manufacturer of the goods; and
(ii) at the time of the importation, the manufacturer of the goods does not have a place of business in
Australia.

3. List at least four different consumer guarantees set out in Part 5-4 of the ACL and
their corresponding statutory sections.

Section 51 ACL: Guarantee that the seller has a right to dispose of the goods.
Section 52 ACL: The consumer has a right to undisturbed possession of the goods.

Section 53 ACL: The goods are free of any undisclosed security, charge or encumbrance.

Section 54 ACL: Suppliers and manufacturers guarantee that goods are of acceptable
quality when sold to a consumer.
Section 55 ACL: The supplier guarantees goods are reasonably fit for any disclosed
purpose for which the supplier represents they are reasonably fit.
Section 56 ACL: If goods are sold by description they must correspond with that description
Section 57 ACL: If goods are sold by sample or demonstration model they correspond with the
sample or model.

Downloaded by ahmad as (ksa-m29@hotmail.com)


lOMoARcPSD|3248894

37
Section 58 ACL: Manufacturers or importers guarantee they will take reasonable steps to provide
spare parts and repair facilities (a place that can fix the consumer’s goods), for a
reasonable time after purchase.

Section 59 ACL: A supplier must not tell a consumer that they are required to pay for any rights
equivalent to a consumer guarantee.
This means that, when selling an extended warranty, a supplier or manufacturer
should be very clear exactly what it offers over and above the consumer
guarantees.
Section 60 ACL: Suppliers guarantee their services are provided with due care and skill.
This means they must:
 use an acceptable level of skill or technical knowledge when
providing the services and
 take all necessary care to avoid loss or damage when providing the
services.
Section 61 ACL: Suppliers guarantee that services will be reasonably fit for any purpose specified
by the consumer and any products resulting from the services are also fit for that
purpose.

Section 62 ACL: A contract or agreement for the supply of services usually states when the
services will be provided and the date they will be completed.
If not, the supplier guarantees they will supply the service within a reasonable
time.

4. Identify when a manufacturer may be liable for breach of the consumer guarantees
set out in Part 5-4 and what liability a manufacturer may have under Part 3-5 of the
ACL.
• Section 259 ACL provides that a person may take action against the supplier of goods for breach of
consumer guarantees.
• When the problem with the good or service is minor, the supplier can choose between providing a
repair or offering the consumer a replacement or a refund.
• When there is a major failure, the consumer can:
– reject the goods or services and either choose a refund or a replacement, or
– ask for compensation for any drop in value of the goods or services.
• Part 3.5 outlines the liability of manufacturers for goods with defects.
• Section 138 ACL provides that a manufacturer of goods is liable to compensate an individual if:
– The manufacturer supplies goods in trade or commerce; and
– The goods have a safety defect; and
– The individual suffers injuries because of the safety defect.
• Section 139 ACL covers liability for loss or damage by a person other than the injured individual.
• Section 140 ACL covers liability for loss or damage to goods if they are destroyed or damaged.
• Section 141 ACL covers liability for loss or damage to land, buildings and fixtures.
• Section 142 ACL outlines possible defences.

5. According to the ACL when do goods have a 'safety defect'?


• Section 9(1) ACL provides that goods have a safety defect if their safety is not such as persons
generally are entitled to expect. The test is objective based on the reasonable expectations of the
community rather than the subjective expectations of the person injured. The question is: Are the
goods as safe as persons generally are entitled to expect?
• For example, in Glendale Chemical Products v ACCC (1998) a person suffered burns to his face and
eyes by an eruption of scalding steam when he tipped caustic soda down a drain containing hot
water. The label on the product stated the product was corrosive and that contact with eyes and skin
should be avoided. The user was advised to wear rubber gloves and safety glasses when handling
the product and to dissolve it in water before pouring it down the drain. It did not contain a warning
not to use hot water. The lack of warning re hot water was a “defect” in the product.

Downloaded by ahmad as (ksa-m29@hotmail.com)


lOMoARcPSD|3248894

38

6. You need a home computer to assist you with your studies at RMIT. After shopping
around, you purchase a computer at Rick Smith for $2000. One month later the
computer screen starts freezing and then the computer turns itself off. The fault is
not caused by a computer virus but is a problem with the actual computer. You
return to Rick Smith and tell them you want your computer replaced.
Pursuant to Commonwealth legislation, on what grounds if any, would Rick Smith
be required to replace the computer?
• ISSUE:
– The legal issue in this matter concerns consumer guarantees, specifically that a product
has to be acceptable quality, under the Australian Consumer Law.
• RULE:
– Section 3(1) ACL provides that a person is taken to have acquired goods as consumer if the
amount paid for the goods does not exceed $40,000; or if the price of the goods exceeds
$40,000, the goods were of a kind ordinarily acquired for personal, domestic or household
use or consumption; or the goods consisted of a vehicle or trailer acquired for use principally
in the transport of goods on public roads.
– Section 54 ACL provides that suppliers and manufacturers guarantee that goods are of
acceptable quality when sold to a consumer.
– Section 54(2) provides that the test is whether a reasonable consumer, fully aware of the
goods’ condition (including any defects) would find them:
• fit for all the purposes for which goods of that kind are commonly supplied
• acceptable in appearance and finish
• free from defects
• safe
• durable

• APPLY:
– The first matter that must be established is that you are a consumer. Here the cost of the
computer is $2,000 (a good which costs less than $40,000) which means you are a consumer
within the definition of s 3 ACL.
– The facts state that the computer starts freezing and turns itself off, and that these faults are
not caused by a virus, but rather the actual computer. This means the computer is not of
acceptable quality, given the defects have emerged one month after purchase. Therefore the
computer is not free from defects: s 54(2)(c) ACL which is a contravention of the consumer
guarantees.
– Major or minor failure?
• CONCLUSION:
– Given that consumer guarantees apply to suppliers and manufacturers, Rick Smith is obliged
to provide you with a replacement computer.

Downloaded by ahmad as (ksa-m29@hotmail.com)


lOMoARcPSD|3248894

39
7. Evan bought a new Blue Ray DVD player from Sales Ltd. The DVD player had been
manufactured overseas and imported by a Victorian company, Quick Imports Pty
Ltd.
After six months of satisfactory operation, the DVD player developed serious faults
and required costly repairs.
Sales Ltd has denied responsibility on the grounds of a term of the contract, signed
by Evan which states:
"It is expressly agreed that the only conditions or warranties expressed or
implied which apply to the contract are those resulting from the warranty
issued by the importer."
The warranty which came with the DVD player read:-
"The importer shall repair any defects up to a period of three months from
the date of purchase. All other conditions and warranties are expressly
excluded."
The importer, Quick Imports Pty Ltd has denied responsibility on the basis of the
warranty.
Assess Evan's legal position and advise him of what action he should take, if
any, under national legislation against Sales Ltd and/or Quick Imports Pty
Ltd.
• Is Evan a “consumer”?
– Evan is a “consumer” under ACL s 3 regardless of the price. Even if the DVD recorder cost >$40K, it is
goods of a kind ordinarily acquired for personal, domestic or household use and is not bought for
resupply or to be used up in trade or commerce
– He therefore has the benefit of the statutory guarantees
• Is the DVD recorder of “acceptable quality” under ACL s 54?
– No, as it is not “fit for all the purposes for which goods of that kind are commonly bought” and is also
not “free from defects” as the DVD recorder had serious faults and needed extensive repairs after six
months’ use
• Is the DVD recorder “fit for its disclosed purpose” under ACL s 55?
– Probably not: Evan has relied on the recommendation of the salesman. If the faults relate to a
purpose that Evan disclosed, the DVD is not fit for a disclosed purpose
– Remedies for non-compliance with the guarantees depend on whether the failure to comply with the
consumer guarantee was a “major failure” as defined in s 260
– Evan may therefore reject the DVD recorder or recover compensation for any reduction in value of the
DVD recorder below the price paid (s 259)
– Evan may also recover damages for any reasonably foreseeable loss or damage (s 259)
– If not a “major failure”, Evan may require Sales Ltd to remedy the failure OR recover all reasonable
remedy costs OR reject the DVD recorder (s 259)
– Evan may therefore recover damages from Sales Ltd for non-compliance with the s 55 guarantee as to
“acceptable quality” and the s 56 guarantee as to ‘fitness for a disclosed purpose”
– The exclusion or limitation clause in favour of the supplier is void under s 64
– Evan may recover damages from Quick Imports for non-compliance with the s 54 guarantee as to
“acceptable quality”: ACL s 271
– The exclusion or limitation clause in favour of the manufacturer is void under s 276

Downloaded by ahmad as (ksa-m29@hotmail.com)


lOMoARcPSD|3248894

40
8. You have just started your marketing business trading as a sole trader. You want to
fit out the premises you have leased. You discuss with a reputable interior designer
that you would like the premises fitted with the retro fixtures you have purchased.
You agree the cost of the fit out will be $1,000. The interior designer affixes light
fittings to the ceilings but does not attach them securely. Consequently two light
fittings fall to the ground and smash. The light fittings were valued at $2,000.
Do you have any legal action against the interior designer under the Australian
Consumer Law?
• ISSUE:
– The legal issue in this matter concerns consumer guarantees, specifically that services
must be provided with due care and skill, under the Australian Consumer Law.
• RULE:
– Section 3(3) ACL provides that a person has acquired services as a ‘consumer’ when the
cost of the service is less than $40,000 or the services were of a kind ordinarily acquired for
personal, domestic or household use or consumption.
– Section 60 ACL provides that suppliers guarantee their services are provided with due care
and skill. This means they must:
– use an acceptable level of skill or technical knowledge when providing the services and
– take all necessary care to avoid loss or damage when providing the services.
• APPLY:
– The first matter that must be established is that you are a consumer. Here the cost of the
interior designer’s service is $1,000 which is less than the $40,000 limit, therefore you are
deemed to be a consumer and the consumer guarantees apply.
– The problem involves the affixing of light fittings by an interior designer which is a service.
Pursuant to s 60 ACL all services must be carried out with due care and skill. The facts state
that the interior designer affixes light fittings to the ceilings but does not attach them securely.
Therefore you could argue that the interior designer has breached the consumer guarantee of
due care and skill under the ACL.
• CONCLUSION:
– The interior designer is required to refund you the $1,000 and pay $2,000 compensation for
the light fittings.

9. It is Christmas and Annie's son Ted keeps asking for an EasyRide Scooter. The
three wheel scooters are very popular and every child seems to have one. Ted is
thrilled to be given an EasyRide Scooter for Christmas and heads out immediately
to ride it along the footpath. Unfortunately one of the front wheels is faulty and falls
off. Consequently Ted flies off the scooter and bangs his head on a pole, knocking
out two of his teeth. Annie has to take time off work to look after her son whilst he is
recovering (happily playing games on his mother's smart phone) in hospital.
The scooter was assembled by ToysRToys Pty Ltd in New South Wales
however many of the components were sourced from China. Investigations have
established that the faulty wheel was defective because of a faulty ball bearing
which ball bearing was made in Korea.
Advise, using statute, whether Annie and Ted are entitled to any
remedies.

• ISSUE:
• The legal issue in this matter concerns safety defects under the Australian Consumer Law.
• RULE:
• Section 9(1) ACL provides that goods have a safety defect if their safety is not such as persons generally are
entitled to expect.
• Manufacturers are liable for loss or damage caused by supply of goods with a ‘safety defect’. This is a strict
liability regime, hence there is no need to prove the manufacturer was negligent. The plaintiff does not need to
be a consumer and the goods need not be consumer goods.
• APPLY:
• The first matter to be addressed is whether the assembler, ToysRToys can be deemed a ‘manufacturer’ of the
EasyRide Scooters. Section 7(1)(a) ACL provides that a manufacturer includes a person who assembles
goods. Given the assembler is located in Australia, they would be the ‘manufacturer’ for ACL purposes.
• The next matter that must be addressed is whether the goods have a ‘safety defect’. This is an objective test.
Arguably, a scooter with a faulty wheel which falls off does not have a safety standard that persons generally
are entitled to expect.
• The safety defect has caused injuries to Ted therefore the manufacturer is liable to compensate him under s
138 ACL. Annie may be able to sue the manufacturer under s 139 ACL for loss to a person other than the
injured individual.
• CONCLUSION:
• ToysRToys is liable as manufacturer to compensate Ted for injuries and Annie for financial loss in taking time
off work, as a consequence of the safety defect in the EasyRide Scooter.
Downloaded by ahmad as (ksa-m29@hotmail.com)
lOMoARcPSD|3248894

41

Week 12 (w/c May 25)


Starting a Business

The following questions relate to the textbook Business Law chapter 10:

1. Why is the choice of business structure an important decision?

2. Compare and contrast the differences between (a) a sole trader, (b) a partnership
and (c) a company.

3. You are about to complete your accounting degree and are talking with an
entrepreneurial fellow student about running a business together upon graduation.
What are some relevant business structure choice factors that may be important to
you if for example your business is a cafe, an executive commercial premises
cleaning service, an accounting firm, an import I export business, a lending
institution?

4. What are the four requirements necessary to satisfy for a business organization to
be a partnership?

5. Shivani and Asher decide to advertise together offering landscaping services.


Shivani already owns the necessary tools and pays the advertising costs. Shivani
and Asher agree that for any customers that Asher finds Shivani will pay her a fee
but that Shivani herself will receive the payments made by the customers.

Are Shivani and Asher in a partnership?

6. You are working as an assistant at a small firm of accountants. A new employee


assistant who has a non-business background has asked you "what is the
doctrine of separate legal entity"? He has to brief a client and requires a short
legal explanation. Please provide him with one.

7. What are the two main ways that companies can be classified?

a. Ryder wants to form a company to provide investment and business finance


services. Advise Ryder generally how a company is established.

9. Do exercise 10.1 on page 551 of the textbook Business Law

10. Do the Quiz on pages 549-551 of the textbook Business Law.

Downloaded by ahmad as (ksa-m29@hotmail.com)

You might also like